Why isn't it B cubed?

Algebra Level 4

1 a 4 + 1 b 4 + 1 c 4 = 1 \dfrac{1}{a^4}+\dfrac{1}{b^4}+\dfrac{1}{c^4}=1

Given the above equation for positive numbers a , b , c a,b,c .

Find the minimum value of

a 4 b 4 + a 4 c 4 + b 4 c 4 a 3 b 2 c 3 \dfrac{a^4b^4+a^4c^4+b^4c^4}{a^3b^2c^3}

If the minimum value of the above is x x , input your answer as 100 x \lfloor 100x \rfloor .

This is part of the set Trevor's Ten

Details and Assumptions

  • The answer is not 300 300 .

  • It is indeed a 3 b 2 c 3 a^3 b^2 c^3 and not a 3 b 3 c 3 a^3 b^3 c^3


The answer is 282.

This section requires Javascript.
You are seeing this because something didn't load right. We suggest you, (a) try refreshing the page, (b) enabling javascript if it is disabled on your browser and, finally, (c) loading the non-javascript version of this page . We're sorry about the hassle.

4 solutions

Trevor Arashiro
Mar 15, 2015

Solution

Simplifying our semi gigantic fraction.

a 4 b 4 + a 4 c 4 + b 4 c 4 a 3 b 2 c 3 = a b 2 c 3 + a c b 2 + b 2 c a 3 \dfrac{a^4b^4+a^4c^4+b^4c^4}{a^3b^2c^3}=\dfrac{ab^2}{c^3}+\dfrac{ac }{b^2}+\dfrac{b^2c}{a^3}

a b 2 c 3 + a c b 2 + b 2 c a 3 = a b 2 c ( 1 a 4 + 1 b 4 + 1 c 4 ) \dfrac{ab^2}{c^3}+\dfrac{ac }{b^2}+\dfrac{b^2c}{a^3}=ab^2c\left(\dfrac{1}{a^4}+\dfrac{1}{b^4}+\dfrac{1}{c^4}\right)

Remember that 1 a 4 + 1 b 4 + 1 c 4 = 1 \dfrac{1}{a^4}+\dfrac{1}{b^4}+\dfrac{1}{c^4}=1 . Thus the above can be simplified to a b 2 c ab^2c .

Thus we are simply trying to minimize a b 2 c ab^2c . By G.M. \leq A.M.

1 4 a 4 b 8 c 4 4 1 a 4 + 1 2 b 4 + 1 2 b 4 + 1 c 4 4 \sqrt[4]{\dfrac{1}{4a^4b^8c^4}}\leq \dfrac{\dfrac{1}{a^4}+\dfrac{1}{2b^4}+\dfrac{1}{2b^4}+\dfrac{1}{c^4}}{4}

Simple algrbra yields.

2 2 a b 2 c 2\sqrt{2}\leq ab^2c

Thus our answer is

2 2 100 = 282 \lfloor 2\sqrt2 \cdot100\rfloor=\boxed{282}

I did it in a basically equivalent way:

Note that the condition is a 4 b 4 + b 4 c 4 + c 4 a 4 = a 4 b 4 c 4 a^4b^4+b^4c^4+c^4a^4=a^4b^4c^4 so we just want to find the minimum of a 4 b 4 c 4 a 3 b 2 c 3 = a b 2 c \dfrac{a^4b^4c^4}{a^3b^2c^3}=ab^2c

Now by GM-HM inequality, 4 a 4 b 8 c 4 4 4 1 a 4 + 1 2 b 4 + 1 2 b 4 + 1 c 4 = 4 \sqrt[4]{4a^4b^8c^4}\ge \dfrac{4}{\dfrac{1}{a^4}+\dfrac{1}{2b^4}+\dfrac{1}{2b^4}+\dfrac{1}{c^4}}=4 so a b 2 c 4 4 4 = 2 2 ab^2c\ge \dfrac{4}{\sqrt[4]{4}}=2\sqrt{2}

Daniel Liu - 6 years, 3 months ago

Log in to reply

Wow, nice alternative. So many solutions, all relatively the same but all with their own unique twist.

Trevor Arashiro - 6 years, 3 months ago

I did the same

Department 8 - 5 years, 6 months ago

i did same!

Dev Sharma - 5 years, 5 months ago

Why is this rated as level 5 with 350 points? Is it that hard? Also, why did it get an official rating after I solved it. Why am I so unlucky?

Prasun Biswas - 6 years, 3 months ago

I don't know what to mention now , since my method has been pretty much covered with what Daniel and Prasun have posted .

Btw your question is indeed genuine , why isn't it B 3 B^{3} ?

@Calvin Lin sir , is there any reason for it ? Sorry for the trouble though .

A Former Brilliant Member - 6 years, 3 months ago

Log in to reply

I'm not sure what u mean. U mean brilliant cubed? Lol

Trevor Arashiro - 6 years, 3 months ago

Log in to reply

Yes , why did the staff choose B^2 ! Just curious now that you have asked it as a question

A Former Brilliant Member - 6 years, 3 months ago

Log in to reply

@A Former Brilliant Member Let B B denote the actions we make on this site. Then B 2 B^2 can be considered as the effect it produces on the community. Now, there are people who produce negative actions on this site (example: people like me). But as you can see, B 2 B^2 always remains non-negative. That is the reason.

Prasun Biswas - 6 years, 3 months ago

Log in to reply

@Prasun Biswas Yeah , the smallest even natural number with that property . Nice explanation :)

A Former Brilliant Member - 6 years, 3 months ago

Is the given condition necessary? I solved it like this which didn't really need the given condition. Here it goes:

The given expression (say ( X ) (X) ) simplifies to,

X = ( a b 2 c 3 + a c b 2 + b 2 c a 3 ) = b 2 ( a c 3 + c a 3 ) + a c b 2 X=\left(\frac{ab^2}{c^3}+\frac{ac}{b^2}+\frac{b^2c}{a^3}\right)=b^2\left(\frac{a}{c^3}+\frac{c}{a^3}\right)+\frac{ac}{b^2}

Applying AM-GM, we have,

a c 3 + c a 3 2 a c X 2 b 2 a c + a c b 2 \frac{a}{c^3}+\frac{c}{a^3}\geq \frac{2}{ac}\implies X\geq \frac{2b^2}{ac}+\frac{ac}{b^2}

Note that RHS of the last inequality will be the tightest lower bound when it is minimized. Minimizing it using AM-GM inequality, we have,

2 b 2 a c + a c b 2 2 2 \frac{2b^2}{ac}+\frac{ac}{b^2}\geq 2\sqrt{2}

Comparing the bounds of the inequalities we obtained, we have,

X 2 2 X\geq 2\sqrt{2}

Hence follows the answer.


Just in case anyone wonders, equality occurs at a = 4 1 / 4 = c , b = 2 1 / 4 a=4^{1/4}=c~,~b=2^{1/4} for all the three inequalities (verification for equality case in AM-GM) and as such, the value we found in the end is indeed the minimum since the equality cases are matched.

Prasun Biswas - 6 years, 3 months ago

Log in to reply

Yes, the condition was not necessary. Note that the inequality we are trying to prove is homogenous.

Daniel Liu - 6 years, 3 months ago

Log in to reply

Yes, I see that. But I don't get what you're trying to imply by that. Does homogeneity have any special kind of impact on inequalities?

Prasun Biswas - 6 years, 3 months ago

Log in to reply

@Prasun Biswas Because the inequality is homogenous (both sides have degree 0) we can let a = k a a'=ka , b = k b b'=kb , and c = k c c'=kc for any non-zero real number k k and yield an identical inequality in a , b , c a',b',c' . However, the condition itself, because it is non-homogeneous, turns into 1 a 4 + 1 b 4 + 1 c 4 = 1 k 4 \dfrac{1}{a^4}+\dfrac{1}{b^4}+\dfrac{1}{c^4}=\dfrac{1}{k^4}

But 1 k 4 \dfrac{1}{k^4} can equal any non-zero real, so we essentially just got rid of the condition.

Daniel Liu - 6 years, 2 months ago

Log in to reply

@Daniel Liu Ah, I get it now. Thanks! :D

Prasun Biswas - 6 years, 2 months ago

I think the conditions are necessary. Because refer to my solution, if I had ( X ) = 2 (X)=2 then the minimum would double

Trevor Arashiro - 6 years, 3 months ago

Log in to reply

Check again! It remains the same. When the given condition is 2 2 , you are also minimizing the double of what you're minimizing now.

Prasun Biswas - 6 years, 3 months ago

Log in to reply

@Prasun Biswas Oh, that's what you meant. I was talking about it changing the answer, not remaining as it is.

Trevor Arashiro - 6 years, 3 months ago

Same method

Figel Ilham - 6 years, 3 months ago

Nice solution! But all inequalities are not proven without the equality case :) although it is quite clear that there must be an equality case.

Joel Tan - 6 years, 3 months ago

Log in to reply

A general result for the problem:

If 1 a 4 + 1 b 4 + 1 c 4 = k , k R + \dfrac{1}{a^4}+\dfrac{1}{b^4}+\dfrac{1}{c^4}=k, k\in\mathbb{R^+} , then equality holds iff,

a = c = α , b = 2 1 / 4 α , α = 2 k 1 / 4 a=c=\alpha~,~b=2^{-1/4}\alpha~,~\alpha=\frac{\sqrt{2}}{k^{1/4}}

Prasun Biswas - 6 years, 3 months ago

Log in to reply

Nicely generalsied

Aakash Khandelwal - 5 years, 7 months ago

And Nice problem.

Is this original?

Dev Sharma - 5 years, 5 months ago
Kelvin Hong
Sep 6, 2018

This problem had been discussed long time ago, but I didn't find solution similar to mine, so below is my solution:

a 4 b 4 + b 4 c 4 + c 4 a 4 a 3 b 2 c 3 = a b 2 c 3 + b 2 c a 3 + 1 2 a c b 2 + 1 2 a c b 2 4 a b 2 c 3 b 2 c a 3 1 2 a c b 2 1 2 a c b 2 4 = 2 2 \begin{aligned}\dfrac{a^4b^4+b^4c^4+c^4a^4}{a^3b^2c^3}&=\dfrac{ab^2}{c^3}+\dfrac{b^2c}{a^3}+\dfrac12\dfrac{ac}{b^2}+\dfrac12\dfrac{ac}{b^2}\\&\geq 4\sqrt[4]{\dfrac{ab^2}{c^3}\cdot\dfrac{b^2c}{a^3}\cdot\dfrac12\dfrac{ac}{b^2}\cdot\dfrac12\dfrac{ac}{b^2}}\\&=2\sqrt2\end{aligned}

I'm surprised that I didn't use the constraint stated in the problem, but I found that the constraint is just narrow down the solution range, this minimum can achieve when a = c = 2 , b = 2 4 a=c=\sqrt2, b=\sqrt[4]2 otherwise we just need a = c = 2 4 b a=c=\sqrt[4]2b so I think there is no need to provide the constraint.

Emmanuel Torres
Feb 9, 2017

I don't know why. but I assumed a^4 = c^4 = 4, and b^4 = 2. And solved from there. There was symmetry between a and c, but not b because of b^2 in the denominator. 32/(8*√2) * 100 = 282.

Aakash Khandelwal
Oct 20, 2015

We need to find minimum value of ab^2c. Now use GM>=HM taking numbers as a^4,2 times 2*b^4 and c^4

0 pending reports

×

Problem Loading...

Note Loading...

Set Loading...